Prove that the complement of a closed set is open

  • Thread starter Thread starter Mr Davis 97
  • Start date Start date
  • Tags Tags
    Closed Set
Click For Summary
The discussion centers on proving that the complement of a closed set is open using a proof by contradiction. It begins by assuming that the complement Sc is not open, leading to the conclusion that for any radius r = 1/n, there exists a point pn in S that is within 1/n of p. This sequence of points pn converges to p, which contradicts the definition of a closed set since p should not be a limit point of S if it is in Sc. The confusion arises around the choice of 1/n and the convergence of the sequence to p, highlighting the relationship between closed and open sets in topology. Ultimately, the proof reinforces the fundamental property that the complement of a closed set is indeed open.
Mr Davis 97
Messages
1,461
Reaction score
44

Homework Statement


Suppose that S is a closed set. We claim that Sc is open. Take any p ∈ Sc. If
there fails to exist an r > 0 such that
d(p, q) < r ⇒ q ∈ Sc
then for each r = 1/n with n = 1, 2, . . . there exists a point pn ∈ S such that
d(p, pn) < 1/n. This sequence in S converges to p ∈ Sc, contrary to closedness of S.
Therefore there actually does exist an r > 0 such that
d(p, q) < r ⇒ q ∈ Sc
which proves that Sc is an open set.

Homework Equations

The Attempt at a Solution


So my question here is not how to do the prove, because I have that proof right here. I am just confused about the proof. I understand that the argument in general is a proof by contradiction, by assuming that Sc is not open and deriving a contradiction. However, I am confused about the reasoning here:

If there fails to exist an r > 0 such that
d(p, q) < r ⇒ q ∈ Sc
then for each r = 1/n with n = 1, 2, . . . there exists a point pn ∈ S such that
d(p, pn) < 1/n. This sequence in S converges to p ∈ Sc.

Can someone explain the logic here? Where does the 1/n comes from? Why does that sequence converge to p?
 
Physics news on Phys.org
You take a point p in the complement where the condition for being an open set does not hold. You then show that there is a sequence in S that converges to that point in S^c, implying that S is not a closed set.

This question is significantly simpler using the general topological definition of a closed set: A closed set is the complement of an open set.
 
Question: A clock's minute hand has length 4 and its hour hand has length 3. What is the distance between the tips at the moment when it is increasing most rapidly?(Putnam Exam Question) Answer: Making assumption that both the hands moves at constant angular velocities, the answer is ## \sqrt{7} .## But don't you think this assumption is somewhat doubtful and wrong?

Similar threads

  • · Replies 5 ·
Replies
5
Views
2K
  • · Replies 5 ·
Replies
5
Views
2K
  • · Replies 24 ·
Replies
24
Views
3K
  • · Replies 3 ·
Replies
3
Views
1K
Replies
8
Views
2K
  • · Replies 1 ·
Replies
1
Views
2K
  • · Replies 1 ·
Replies
1
Views
1K
  • · Replies 13 ·
Replies
13
Views
2K
  • · Replies 14 ·
Replies
14
Views
2K
Replies
5
Views
2K